ChaseDream
搜索
返回列表 发新帖
楼主: UlysessHope

[逻辑小分队] 【每日逻辑练习第二季】【1-17】

[复制链接]
发表于 2012-10-18 21:10:15 | 显示全部楼层
精练(37'7)
Bg:Medical report result
pre1: babies born prematurely were more likely to have low birth weights and to
     suffer from health  problems than were babies not born prematurely
pre2:mothers who had received adequate prenatal care were less likely to have
    low birth weight babies than were mothers who had received inadequate prenatal care
con:Adequate prenatal care significantly decreases the risk of low birth weight babies
(weaken)
Pre:C             E
   ADP         DECREASE THE RISK OF LOW B.W babies
   C--->E but it is not a necessary informatio.There is another effect/
Ans:B 因为文中只说了她们被送进来之后的情况,可是没有说明送进来之前是怎么样的,数据会失真
The key (解析)The answer choice falls into the category of “Showing a
statistical problem exists with the data used to make the causal statement.”
A--support
C--Can not effect .It's only a conclusion
D--support.restate the conculsion.---如果对这个答案取非,答案就会出问题了
E--POSITIVE ANSWER
逻辑链65(21'2)
pre1:Sales of telephones have increased dramatically over the last year
pre2:extensive advertising of M.I.'S product
con:M.I.plans to expand production of its own model of telephone
(Cannot increase??这种应该是什么样的题型啊.....)
PRE:The M.I.company understand this increasion oneside.The boss don't know that the sales of telephones have increased dramatically over the last year .However, this year can not be predictated.And ade.'s effect is limited .
Ans:A---CORRECT E
知道正确答案后我的分析:
A--support 毕竟它的手机都卖完了,到达了它想要拓展它的产量的目的,因为说明它的产品还是有一定得地位的
B--也是一个support的答案,库存少了 说明卖的好了
C--这个和题中的条件是矛盾的
D--support/irrelevant
先占个位置,要上课了.........哎
发表于 2012-10-20 15:49:49 | 显示全部楼层
精炼
weaken 39s
premature baby tend to have less weight
&mothers who had received adequate prenatal care --less likely have that knd of baby
conclusion:adequte p care--decrease less weight baby
my opinion:having premature baby has other reasons
choice:C
A stenghten
B so what
C correct
D not convincing
E irrelevant
-------------------------------------------------------------------------------------
逻辑链
1.support 20s
last yr telephone sales increase,in order to increase--extend production & continuing adds
my opinion: too much adds fees will not increase the sales.
choice:E
A irrelevant
B irrelevant
C irrelevant
D not useful
E correct
2.assumption  29s
background:depositor protected by the government
premise:insurance made customers less likely to choose
conclusion:insurance is only partly responsible
my assumption:the customers have the ability to choose
choice:E
A irrelevant
B irrelevant
C irrelevant
D not convincing enough
E correct
3.weaken 25s
a type of tree can be made of drug--extinction
my opinion:if the tree can be cultivated,then the problem is solved
choice
A irrelevant
B irrelevant
C irrelevant
D correct
E not useful,man is so great
4.support 28s
manufacturers discount price when it's advertising--increase bought by retailers
but more profit will be hold by manufacturers by not doing that.
my opinion:maybe because the product it sells is necessary everyday.
choice
A irrelevant
B irrelevant
C not useful
D correct
E weaken
发表于 2012-10-20 16:02:50 | 显示全部楼层
纠错:精练题又错了,这次错的最不可原谅,我好像是烦了。
4. Medical researcher: As expected, records covering
the last four years of ten major hospitals
indicate that babies born prematurely were
more likely to have low birth weights and to
suffer from health problems than were babies
not born prematurely. These records also
indicate that mothers who had received
adequate prenatal care were less likely to have
low birth weight babies than were mothers who
had received inadequate prenatal care.
Adequate prenatal care, therefore, significantly
decreases the risk of low birth weight babies.
Which one of the following, if true, most weakens the
medical researcher’s argument?
(A) The hospital records indicate that many babies
that are born with normal birth weights are
born to mothers who had inadequate prenatal
care.
(B) Mothers giving birth prematurely are routinely
classified by hospitals as having received
inadequate prenatal care when the record of
that care is not available.
(C) The hospital records indicate that low birth
weight babies were routinely classified as
having been born prematurely.

还是老话,如果有更有说服力的选项存在,纠结于细节的小错误就大可不必了。C 虽然在题干里说的是低体重的孩子常常是早产儿,但是等同这两个概念是无可厚非的。
B 如果医院将体重不够的孩子不经检查统统划归为早产,那就非常有力的打击了题干的结论和逻辑。
启示:我好像真的安排错复习顺序了,我应该先开始阅读训练的。
发表于 2013-1-16 11:23:02 | 显示全部楼层
4.

Medical researcher: As expected, records covering
the last four years of ten major hospitals
indicate that babies born prematurely were
more likely to have low birth weights and to
suffer from health problems than were babies
not born prematurely. These records also
indicate that mothers who had received
adequate prenatal care were less likely to have
low birth weight babies than were mothers who
had received inadequate prenatal care.
Adequate prenatal care, therefore, significantly
decreases the risk of low birth weight babies.

In this argument, there is more than one logical line, thus making it difficult to understand. Firstly, the researcher points out babies born prematurely are more likely to have low birth weights than babies not born prematurely. Secondly, the argument reasons that since mothers who have received inadequate prenatal care are more likely to have low birth weight babies than mothers who have received adequate prenatal care, adequate prenatal care, therefore, significantly decreases the risk of low birth weight babies. How to weaken the argument? Since the conclusion is about relationship between adequate prenatal care and the risk of low birth weight babies, if we can break the relationship, we’ll solve the problem. So the answer should refer to adequate prenatal care and low birth weight babies.

这道题和OG里面有一道关于结婚和长寿关系的题非常类似,其中也引入了如吸烟等不良习惯等因素使得逻辑关系比较复杂。但是,需要注意结论是什么,把相关关系找出来,从而逐一击破!
Which one of the following, if true, most weakens the
medical researcher’s argument?

(A) The hospital records indicate that many babies
that are born with normal birth weights are
born to mothers who had inadequate prenatal
care.

It is normally that some mothers who had inadequate prenatal care would have babies with normal birth weights, thus not weakening the argument.
(B) Mothers giving birth prematurely are routinely
classified by hospitals as having received
inadequate prenatal care when the record of
that care is not available.

Firstly, the answer choice connects low birth weight babies, whose mothers gave birth prematurely, with prenatal care. So it seems to be valid. What’s more, the answer choice points out that in this case, the hospitals increase the mothers who received inadequate prenatal by adding mothers giving birth prematurely when the record of that care is not available. Thus, the real situation may be that not so much inadequate prenatal care contributes to the low birth weight babies. BA

This answer choice falls in the category of “Showing a statistical problem exists with the data uses to make the causal statement.”
(C) The hospital records indicate that low birth
weight babies were routinely classified as
having been born prematurely.


At first glance, it seems like a contender. But the choice actually has no effect on the conclusion, which refers to prenatal care and low birth weight babies.
(D) Some babies not born prematurely, whose
mothers received adequate prenatal care,
have low birth weights.

The situation is similar to A.
(E) Women who receive adequate prenatal care are
less likely to give birth prematurely than are
women who do not receive adequate prenatal
care.

Actually, this choice strengthens the argument, rather than weakens.



65. (35041-!-item-!-188;#058&007595)
Sales of telephones have increased dramatically over the last year. In order to take
advantage of this increase, Mammoth Industries plans to expand production of its own model
of telephone, while continuing its already very extensive advertising of this product.
Which of the following, if true, provides most support for the view that Mammoth Industries
cannot increase its sales of telephones by adopting the plan outlined above?

(A) Although it sells all of the telephones that it produces, Mammoth Industries' share of all
telephone sales has declined over the last year.

It seems to be contender. However, even though the Mammoth Industries’ share has declined over the last year, the total sales of MI can increase.
(B) Mammoth Industries' average inventory of telephones awaiting shipment to retailers has
declined slightly over the last year.

Actually, since MI’s average inventory declined slightly, it actually strengthens the argument.
(C) Advertising has made the brand name of Mammoth Industries' telephones widely known,
but few consumers know that Mammoth Industries owns this brand.

It does point out a weakness in the advertising, but it has no relationship with the sales increase. I wonder a better explanation.

A more thorough analysis reveals that the answer choice is irrelevant, because the fact whether consumers know that MI owns this brand, considering MI’s telephones widely known, has no effect on the argument.
(D) Mammoth Industries' telephone is one of three brands of telephone that have together
accounted for the bulk of the last year's increase in sales.

It cannot weaken the argument.
(E) Despite a slight decline in the retail price, sales of Mammoth Industries' telephones have
fallen in the last year.

Yes, it makes clearly that although the price declined, sales of MI’s telephones still fell in last year. BA



66. (35089-!-item-!-188;#058&007596)
Bank depositors in the United States are all financially protected against bank failure because
the government insures all individuals' bank deposits. An economist argues that this
insurance is partly responsible for the high rate of bank failures, since it removes from
depositors any financial incentive to find out whether the bank that holds their money is
secure against failure. If depositors were more selective, then banks would need to be
secure in order to compete for depositors' money.
The economist's argument makes which of the following assumptions?

(A) Bank failures are caused when big borrowers default on loan repayments.

Although the argument points out that the government insure is partly responsible for the high rate of bank failures, the reason of bank failures is actually irrelevant.
(B) A significant proportion of depositors maintain accounts at several different banks.

The data has no effect on the argument.
(C) The more a depositor has to deposit, the more careful he or she tends to be in selecting
a bank.

This is true, but we do not know the ability of depositors to choose the secure bank.
(D) The difference in the interest rates paid to depositors by different banks is not a
significant factor in bank failures.

The same to A.
(E) Potential depositors are able to determine which banks are secure against failure.

Yes, in this case, depositors become more selective, and will make a more reasonable choice, thus forcing banks to compete for depositors’ money. On the other hand, if we use “negation”, we’ll find that the failure to determine which banks are secure against failure will make the conclusion fallacious. BA

67.前面的作业已经出现过了



68. (35185-!-item-!-188;#058&007601)
Manufacturers sometimes discount the price of a product to retailers for a promotion period
when the product is advertised to consumers. Such promotions often result in a dramatic
increase in amount of product sold by the manufacturers to retailers. Nevertheless, the
manufacturers could often make more profit by not holding the promotions.
Which of the following, if true, most strongly supports the claim above about the
manufacturers' profit?

(A) The amount of discount generally offered by manufacturers to retailers is carefully
calculated to represent the minimum needed to draw consumers' attention to the product.

Since the minimum needed to draw consumers’ attention to the product, we may think that if the amount of discount increases slightly, the sales would increase slightly, thus weakening the argument.
(B) For many consumer products the period of advertising discounted prices to consumers is
about a week, not sufficiently long for consumers to become used to the sale price.

It seems to be a contender, but a more thorough analysis reveals that it is a Shell Game answer. The argument refers to whether discount causes more profit, having no relationship with consumers who become used to the sale price. Even though a week is not sufficiently long for consumers, the sales could still increase, thus increasing total profit.
(C) For products that are not newly introduced, the purpose of such promotions is to keep
the products in the minds of consumers and to attract consumers who are currently using
competing products.

It still points out the advantage of the promotion, thus not weakening the argument.
(D) During such a promotion retailers tend to accumulate in their warehouses inventory
bought at discount; they then sell much of it later at their regular price.

In this case, we can see that such promotions do not cause consumers to buy more products, but cause retailers to buy more. The real benefits are obtained by retailers, rather than manufactures. So there is no need to make such promotions. BA
(E) If a manufacturer fails to offer such promotions but its competitor offers them, that
competitor will tend to attract consumers away from the manufacturer's product.

In fact, this answer choice weakens the argument.
发表于 2013-2-21 14:40:18 | 显示全部楼层
3:20s
b
premise:
premature baby --->low weight
mother inadequate care--->low weight
conclusion:
mother with adequate care ----decrease the possibility of low weight
prephrase:maybe adecate care but premature cause?

A) The hospital records indicate that many babies
that are born with normal birth weights are
born to mothers who had inadequate prenatal
care.---------reverse fact can do nothing with the conclusion

(B) Mothers giving birth prematurely are routinely
classified by hospitals as having received
inadequate prenatal care when the record of
that care is not available.-----------correct

(C) The hospital records indicate that low birth
weight babies were routinely classified as
having been born prematurely.---------------the conclusion has relation with low weight and  inadequate care
(D) Some babies not born prematurely, whose
mothers received adequate prenatal care,
have low birth weights.-------some cases is persuading to the conclusion
(E) Women who receive adequate prenatal care are
less likely to give birth prematurely than are
women who do not receive adequate prenatal
care.--------strengthen
发表于 2013-2-21 15:01:19 | 显示全部楼层
2:02s
c
premise:plan to expand the sales and continue the extensive ads
conclusion: increase the sale
rephrase:maybe the effect of the methods is not so promising


2:05s
e
premise:government makes the assurance that they will responsible for deposits
       the assurance cause banker careless with the depositor
conclusion:the assurance cause failure of bank

59s weaken
d
premise: the drug need  a lot of bark of  the tree
conclusion: the tree will extinct
rephrase:maybe the bark can recreation

1:25s support
premise:promtion period, price discount to retalers may increase the sale amount
conclusion: not promotion the profit grow
rephrase:maybe the retailer sold at the same price whether promotion or not
b

看来我还是不能图快,要先能分清逻辑链才行
发表于 2013-3-20 19:17:04 | 显示全部楼层
46" (Weak)
P: Record indicates: 1. born prematurely babies r more likely to have low weight, 2. moms who received adequate prenatal care r less likely to have low weight baby.
P:
C: prenatal care significantly decrease the risk of low birth weight babies.
Pre: moms who receive adequate prenatal care r less likely to birth baby prematurely.??    less likely ≠ significantly

Answers:
发表于 2013-7-14 13:54:26 | 显示全部楼层
支持支持
发表于 2013-9-2 07:08:07 | 显示全部楼层
5'47''CEEDD
发表于 2013-9-20 16:29:33 | 显示全部楼层
B EEDD
1. 32'
P: prematurely≈low birth weight         maturely≈normal birth weight
    inadequate prenatal care≈low birth weight     adequate prenatal care≈normal birth weight
C: adequate prenatal care significantly decreases the risk of low birth weight babies
Weaken: adequate prenatal≠maturely
答案:B
(A) The hospital records indicate that many babies that are born with normal birth weights are born to mothers who had inadequate prenatal care.-----------"many" is too vague to be useful. the truth about inadequate prenatal does not affect the truth of adequate prenatal.
(B) Mothers giving birth prematurely are routinely classified by hospitals as having received inadequate prenatal care when the record of that care is not available.----------CORRECT. means not available≠inadequate prenatal. maybe has adequate prenatal but also give birth prematurely.
(C) The hospital records indicate that low birth weight babies were routinely classified as having been born prematurely.------------this statement does not illustrate the relationship between adequate prenatal and normal weight.
(D) Some babies not born prematurely, whose mothers received adequate prenatal care, have low birth weights.--------"some" is too vague to be justified.
(E) Women who receive adequate prenatal care are less likely to give birth prematurely than are women who do not receive adequate prenatal care.--------------support. definitely means adequate=normal weight

2. 20'
BG: Sales of telephones have increased dramatically over the last year
P:  expand production of its own model of telephone, while continuing its already very extensive advertising of this product
C: to take advantage of this increase≈increase telephones sales.
CANNOT INCREASE ITS SALES: advertising is useless. other increase≠MI will also increase
答案:E
(A) Although it sells all of the telephones that it produces, Mammoth Industries' share of all telephone sales has declined over the last year.----------------------share decline does not mean amount decline. perhaps the market broaden.
(B) Mammoth Industries' average inventory of telephones awaiting shipment to retailers has declined slightly over the last year.----------inventory decline means more sales. and the awaiting shipment is irrelevant
(C) Advertising has made the brand name of Mammoth Industries' telephones widely known, but few consumers know that Mammoth Industries owns this brand.-------------whether customers know MI owns the brand does not affect the widely known of MI's telephone. irrelevant
(D) Mammoth Industries' telephone is one of three brands of telephone that have together accounted for the bulk of the last year's increase in sales.-----------support the plan
(E) Despite a slight decline in the retail price, sales of Mammoth Industries' telephones have fallen in the last year.-------------CORRECT. means the promising overall market situation does not bring better situation for MI. thus the plan will not succeed.

3. 29'
P: high rate of bank failures is due to government insurance of individuals' bank deposits
C: if depositors were more selective, banks would need to be secure≈low rate of bank failure
ASSUMPTION: depositors can distinguish the safer bank
答案:E
(A) Bank failures are caused when big borrowers default on loan repayments.-------------how the bank fail is not discussed. should focus on the depositors
(B) A significant proportion of depositors maintain accounts at several different banks.------------if not, this statement does not affect the conclusion.
(C) The more a depositor has to deposit, the more careful he or she tends to be in selecting a bank.----------when the depositor begin to select is not the reasoning of this question. and "the more" here just specify one situation, not general, not good.
(D) The difference in the interest rates paid to depositors by different banks is not a significant factor in bank failures.------------we are not told the factors that affect the bank failures
(E) Potential depositors are able to determine which banks are secure against failure.-------------CORRECT. if not able to determine, then more selective does not mean more safer.

4. 16'
P: It takes the bark of 5,000 trees to make one kilogram of the drug
C: continued production of the drug must inevitably lead to the ibora's extinction.
Weaken: highly used≠extinction, maybe more cultivation
答案:D
(A) The drug made from ibora bark is dispensed to doctors from a central authority.----------how the drug is dispensed is not matter. this statement does not state the different of different outcome
(B) The drug made from ibora bark is expensive to produce.-----------what is the cost does not affect the production. we just not consider the price.
(C) The leaves of the ibora are used in a number of medical products.-------------just states the use of leaves, but nothing about the extinction
(D) The ibora can be propagated from cuttings and grown under cultivation.-----------CORRECT.means although cut down, but still have cultivation, extinction will not emerge.
(E) The ibora generally grows in largely inaccessible places.---------------where it grows does not matter.

5. 21'
P: Such promotions often result in a dramatic increase in amount of product sold by the manufacturers to retailers
C: manufacturers could often make more profit by not holding the promotions
SUPPORT: the promotions do not generate more profit
答案:D
(A) The amount of discount generally offered by manufacturers to retailers is carefully calculated to represent the minimum needed to draw consumers' attention to the product.---------means the promotion will cause attention, slightly imply more sales and more profits
(B) For many consumer products the period of advertising discounted prices to consumers is about a week, not sufficiently long for consumers to become used to the sale price.--------------what does "many consumer products" mean? this just suggests longer promotion time, but does not state anything about the advantages of not holding the promotion
(C) For products that are not newly introduced, the purpose of such promotions is to keep the products in the minds of consumers and to attract consumers who are currently using competing products.----------what the promotion of newly introduced products aims is irrelevant. and this just state the essential part of the promotion.
(D) During such a promotion retailers tend to accumulate in their warehouses inventory bought at discount; they then sell much of it later at their regular price.------------CORRECT. the promotion just increase the inventory but not profits. and retailers profits most during this kind of promotion. If without this promotion, manufacturers can profit more.
(E) If a manufacturer fails to offer such promotions but is competitor offers them, that competitor will tend to attract consumers away from the manufacturer's product.----------states the importance of the promotion. thus weaken the claim.
您需要登录后才可以回帖 登录 | 立即注册

Mark一下! 看一下! 顶楼主! 感谢分享! 快速回复:

手机版|ChaseDream|GMT+8, 2024-3-29 08:51
京公网安备11010202008513号 京ICP证101109号 京ICP备12012021号

ChaseDream 论坛

© 2003-2023 ChaseDream.com. All Rights Reserved.

返回顶部